LSAT and Law School Admissions Forum

Get expert LSAT preparation and law school admissions advice from PowerScore Test Preparation.

User avatar
 Dave Killoran
PowerScore Staff
  • PowerScore Staff
  • Posts: 5853
  • Joined: Mar 25, 2011
|
#40820
Complete Question Explanation
(The complete setup for this game can be found here: lsat/viewtopic.php?t=15545)

The correct answer choice is (D)

A glance at the answer choices to this question shows that the answers are numerical in nature. Thus, refer to the distributions to answer this question.

If F and S are assigned to the same floor, the 3-3-1 must be in effect (F and S are assigned to different floors in the 4-2-1). This information immediately eliminates answer choices (B), (C), and (E), each of which feature exactly 2 or 4 departments on a floor.

In the 3-3-1, L would still be on the top or bottom floor (again, due to the interaction of the HI block and L), and this information then eliminates answer choice (A).

Accordingly, answer choice (D) is correct.

Get the most out of your LSAT Prep Plus subscription.

Analyze and track your performance with our Testing and Analytics Package.